Munkres - Topology - Chapter 1 Solutions: Section 3

You might also like

Download as pdf or txt
Download as pdf or txt
You are on page 1of 16

Munkres - Topology - Chapter 1 Solutions

Section 3
Problem 3.2. Let C be a relation on a set A. If A0 ⊆ A, define the restriction of C to A0 to be the relation C ∩ (A0 × A0 ).
Show that the restriction of an equivalence relation is an equivalence relation.

Solution: Let C 0 be the restriction of C to A0 . As an initial matter, clearly if (a, b) ∈ C 0 , then (ab ) ∈ C. Further, if
(a, b) ∈ C and a, b ∈ A0 , then (a, b) ∈ C 0 . We will now show that the C 0 is an equivalence relation.
Let x0 ∈ A0 . Since (x0 , x0 ) ∈ C, it follows that (x0 , x0 ) ∈ C 0 . Therefore C 0 is reflexive.
Let (x1 , y1 ) ∈ C 0 . It follows that (y1 , x1 ) ∈ C, so (y1 , x1 ) ∈ C 0 . Therefore, C 0 is symmetric.
Let (x2 , y2 ), (y2 , z2 ) ∈ C 0 . It follows that (x2 , z2 ) ∈ C, so (x2 , z2 ) ∈ C 0 . Therefore, C 0 is transitive. Since all the
necessary properties are met, C 0 is an equivalence relation.

Problem 3.4. Let f : A → B be a surjective function. Let us define a relation on A by setting a0 a1 if f (a0 ) = f (a1 ). (a)
Show that this is an equivalence relation. (b) Let A∗ be the set of equivalence classes. Show there is a bijective correspondence
of A∗ with B.

Solution: Part (a) Suppose R is the relation of interest. Let a0 ∈ A. Clearly f (a0 ) = f (a0 ), so a0 ∼ a0 . It follows that
R is reflexive. Now let a0 , a1 ∈ A where a0 ∼ a1 . Since f (a0 ) = f (a1 ) implies that f (a1 ) = f (a0 ), we infer that a1 ∼ a0 .
It follows that R is symmetric. Finally, let a0 , a1 , a2 ∈ A where a0 ∼ a1 and a1 ∼ a2 . We see that f (a0 ) = f (a0 ) = f (a2 ),
so a0 ∼ a2 . Therefore, R is transitive. Accordingly, A is an equivalence relation.
Part (b) Let A be the collection of equivalence classes of A of R, where if x ∈ A, there is an Ex ∈ A such that
Ex = {y ∈ A : x ∼ y}. Let g : A → B where g(Ex ) = f (x). First we will show that g is injective. Let Ex , Ey ∈ A and
g(Ex ) = g(Ey ). It follows that f (x) = f (y), so x ∈ Ex and x ∈ Ey . Since Ex and Ey are not disjoint, we infer from
Lemma 3.1 that Ex = Ey . Therefore, g is injective.
Now let b ∈ B. Since f is surjective, there is an a ∈ A such that f (a) = b. Therefore, g(Ea ) = f (a) = b. Accordingly,
g is surjective, from which it follows that g is a bijection from A to B.

Problem 3.10. (a) Show that the map f : (−1, 1) → R of Example 9 is order preserving. (b) Show that the equation
g(y) = 2y/[1 + (1 + 4y 2 )1/2 ] defines a function g : R → [−1, 1] that is both a left and a right inverse for f .

Solution: Part (a) Let A = (−1, 1). The function f is order preserving if a0 < a1 implies that f (a0 ) < f (a1 ) for all
a0 , a1 ∈ A. Now let x, y ∈ A. We may assume without loss of generality that x < y. We then have:
x−y
f (x) − f (y) = < 0,
(1 − x2 )(1 − y 2 )
because 1 − x2 , 1 − y 2 > 0 for all x, y ∈ A. It follows that f (x) < f (y). The function f is therefore order preserving.
Part (b) This problem a straightforward (but annoying) substitution of f and g into each other. let x ∈ A. We then
have:
  
2x 1 2x
(g ◦ f )(x) = = = x,
1 − x2 1 + [1 + 4x2 /(1 − x2 )2 ]1/2 (1 − x2 ) + (1 + x2 )
showing that g is a left inverse of f .
For (f ◦ g), we can again do the substitution and show (through a lot of algebra) that (f ◦ g)(y) = y. For y ∈ R, we
have:

2y(1 + (1 + 4y 2 )1/2 )
  
2y 1
(f ◦ g)(y) = 2 1/2 2 2 1/2 2
= = y,
1 + (1 + 4y ) 1 − 4y /[1 + (1 + 4y ) ] 2(1 + (1 + 4y 2 )1/2 )
showing that g is a right invoice for f .

Problem 3.11. Show that an element in an ordered set has at most one immediate successor and at most one immediate
predecessor. Show that a subset of an ordered set has at most one smallest element and at most one largest element.
Solution: First we will show that an element in an ordered set has at most one immediate successor. Let A be an ordered
set and x ∈ A. If x is the largest element of A, then it has no immediate successor. Now suppose instead that x has
two immediate successors y, z ∈ A. If we assume y 6= z, we may further assume without loss of generality that y < z.
Since each is an immediate successor of x, by definition (x, y) = (x, z) = ∅. However, since x < y < z, it must be that
y ∈ (x, z), which is a contradiction. Therefore, y = z, establishing that any element in an ordered set has at most one
immediate successor. A similar argument shows that any element of A has at most one immediate predecessor.
Now we will show that A has at most one smallest element. Clearly A may not have any smallest element (for example,
the empty set or {x ∈ R : x > 0}). Suppose x0 , x1 ∈ A are its two smallest elements. By definition, if y ∈ A, then x0 ≤ y
and x1 ≤ y. We infer then that x0 ≤ x1 and x1 ≤ x0 . It follows that x0 = x1 . Accordingly, A has at most one smallest
element. A similar argument shows that A has at most one largest element.

Problem 3.13. Prove the following: Theorem. If an ordered set A has the least upper bound property, then it has the greatest
lower bound property.

Solution: Let A0 be a non-empty subset of A that has a lower bound in A, and let B be the set of all elements of A that
are lower bounds of A0 (which is plainly non-empty). It follows that for each b ∈ B, if a ∈ A0 then b ≤ a. The set B
is therefore bounded above by the elements of A0 . Since A has the least upper bound property, it follows that B has a
supremum x ∈ A. It follows that if x0 ∈ B, then x0 ≤ x, so x is at least as large as any lower bound of A0 . We now need
to show that x is a lower bound of A0 . Let a ∈ A0 . Since x = sup B, it cannot be that a < x; otherwise, x could not be
the supremum of A0 because a ≥ b for all b ∈ B. Accordingly, x = inf A0 .
Since every non-empty subset of A with a lower bound in A has an infimum, it follows that A has the greatest lower
bound property.
Following an analogous argument, we can show that if A has the greatest upper bound property, then it has the least
upper bound property.

Problem 3.14. If C is a relation on a set A, define a new relation D on A by letting (b, a) ∈ D if (a, b) ∈ C. (a) Show that
C is symmetric if and only if C = D. (b) Show that if C is an order relation, then D is also an order relation. (c) Prove the
converse of the theorem in Exercise 13.

Solution: In all my solutions, I assume that D = {(b, a) ∈ A × A : (a, b) ∈ C}. The prompt isn’t entirely clear on this
(there is no ”if and only if”).
Part (a) Suppose C is symmetric. If (a, b) ∈ C, then (b, a) ∈ C. Therefore, (a, b) ∈ D, so C ⊆ D. If (b, a) ∈ D, then
(a, b) ∈ C, from which it follows that (b, a) ∈ C. Therefore, D ⊆ C, so C = D.
Now suppose C = D. If (a, b) ∈ C, then (b, a) ∈ D, from which it follows that (b, a) ∈ C. Therefore, C is symmetric.
Part (b) Assume C is an order relation. Since (a, a) ∈ / C, it follows that (a, a) ∈
/ D. Now let a, b ∈ A. Either (a, b) ∈ C
or (b, a) ∈ C, but not both. Therefore, either (a, b) ∈ D or (b, a) ∈ D, but not both. Now suppose (a, b), (b, c) ∈ C. It
follows (a, c) ∈ C. Therefore, (c, b), (b, a), (c, a) ∈ D. With all the properties met, D is an order relation.
Part (c) Let C be an order relation on A. Let D be an order relation on A where D = {(b, a) ∈ A × A : (a, b) ∈ C}.
Let A0 be a subset of A. Observe that any upper bound of A0 by C is a lower bound of A0 by D. Suppose x ∈ A is an
upper bound of A0 by C, in which case if a ∈ A0 , then a ≤C x. It follows that x ≤D a, so x is a lower bound of A0 .
There is an analogous result for the suprema and infima under C and D. Suppose x0 = sup A0 by C. As just shown, x0
is a lower bound of A0 by D. If x00 is any upper bound of A0 by C, then x0 ≤C x00 , from which it follows that x00 ≤D x0 .
Therefore, x0 = inf A0 by D.
Now suppose A have the greatest lower bound property by D. Therefore, A has the least upper bound property by C.
Applying the result in Exercise 3.13, we infer that A has the greatest lower bounded property by C. But this means that
A has the least upper bound property by D. Accordingly, if A has the greatest lower bound property, it also has the least
upper bound property.

Section 4
Problem 4.1. [See question.]

Page 2
Solution: I only show solutions to some of the more interesting problems here. To be true to the problem, I work out the
steps (particularly the distributive and commutative properties of R) in special detail.
Part (c) By (4) there is a −0 such that 0 − 0 = 0. It follows from (1), (2), and (4) that:

−0 = −0 + 0 = 0 − 0 = 0.

Part (d) By (4), for x ∈ R, there is a −x such that x − x = 0. Applying (4) again, there is a −(−x) such that
−x − (−x) = 0. Putting these together with (1) and (3):

(x − x) − (−x) = x + (−x − (−x)) = x = 0 − (−x) = −(−x) + 0 = −(−x).


Part (e) For x, y ∈ R, for xy there is a −xy by (4). Since x · 0 = x(y − y) = 0, we have from (1), (2), and (5):

x(y − y) − xy = (xy + x(−y)) − xy = x(−y) + (xy − xy) = x(−y) = 0 − xy = −xy + 0 = −xy.

Following the same argument, since y · 0 = 0 · y = (x − x) · y = 0, we have:

(x − x)y − xy = (xy + (−x)y) − xy = (−x)y + (xy − xy) = (−x)y = 0 − xy = −xy.

It follows that x(−y) = (−x)y = −xy.


Part (f) Applying part (e) and (2), we have x(−y) = (−y)x = (−x)y. Plugging in y = −1 and using (4), we have
(−1)x = −x(1) = −x.
Part (i) There is a 1/x such that 1/x · x = x/x = 1.
Part (k) From exercise (j), 1/1 = 1. Therefore:
1 x
x=x·1=x· = .
1 1
Part (l) Because x 6= 0 and y =
6 0, there are multiplicative inverses 1/x and 1/y by (4). As a result, by (1) and (2):

(xy)(1/x · 1/y) = x · (y · (1/x · 1/y)) = x · (1/x · (y · 1/y)) = x · (1/x · 1) = x · 1/x = 1.


It cannot be that xy = 0 because, if it were, the foregoing expression would equal zero by exercise (b). Therefore,
xy 6= 0.
Part (m) By exercise (l), yz 6= 0, so it has multiplicative inverse 1/(yz) by (4). By (1) and (2), we have:
    
1 1 1 1
1·1= y· z· = (yz) · = 1.
y z y z
It follows that (1/y)(1/z) is a multiplicative inverse of yz. But 1/(yz) is also a multiplicative inverse of yz, and by (4)
the inverse is unique. We infer then that (1/y)(1/z) = 1/(yz).
Part (n) From exericse (m) and properties (1) and (2), we have:
       
x w 1 1 11 1 xw
= x· w· = (xw) = (xw) · = .
y z y z yz yz yq

Part (o) From exercise (j), y/y = z/z = 1. From (1), (2), and (5) and exercise (m), we have:
   
x w x w x z w y 1 1 1 1
+ = · 1 + · 1 = · + · = (xz) · + (wy) ·
y z y z y z z y y z z y

1 1 1 xz + wy
= (xz) + (wy) = (xz + wy) = .
yz yz yz yz
Part (p) There is a 1/x by (4). Since 1/x · x = 1, it must be that 1/x 6= 0; otherwise, by exercise (b) 1/x· = 0, a
contradiction.
Part (q) Since z 6= 0, it follows from (4) and exercise (p) that 1/z exists and is non-zero. Since w 6= 0, we infer
that w · 1/z = w/z 6= 0 from exercise (l). Applying (4), we see that w/z has multiplicative inverse (1/(w/z). Since
w/z · 1/(w/z) = 1, we have from (1) and (2) and exercises (j) and (n):
    
z w 1 z w 1 1 1
· · = · · = (wz)
w z w/z w z w/z wz w/z
wz 1 1 z z
= · = = ·1= .
wz w/z w/z w w

Page 3
Solution: Part (r) From exercises (n) and (q):

x/y x z xz
= · = .
w/z y w yw

Part (s) From (1):      


ax 1 1 x
= (ax) =a x· =a .
y y y y
Part (t) From exercises (f) and (s), (−x)/y = [(−1)(x)]/y = (−1)(x/y) = −(x/y).
The multiplicative inverse of −y is 1/(−y). Starting with (−y) · 1/(−y) = 1, we then have from (1) and (2) and
exercise (d) and (g):
          
−x 1 −x 1 1 1 1 1
(−y) · = (−1)(y) · = (−1)(−x) · y· = x· y·
y −y y −y y −y y −y
     
1 1 1 1 x −x −x
= ·x y· = ·y x· = = ·1= .
y −y y −y −y y y
It follows that (−x)/y = x/(−y) = −(x/y).

Problem 4.2. [See question.]

Solution: Part (a) From (6), x + z > y + z. Relying on (4) and (6), we see that w + x = x + w > z + x = x + z. By
the transitivity property of order relations, x + w > y + z.
Part (b) From exercise 2(a), x + y > 0 + 0 = 0.
Part (c) Suppose x > 0. Applying (6), we have x + (−x) = 0 > 0 + (−x) = −x. Conversely, suppose −x < 0. We
then have −x + x = 0 < 0 + x = x.
Part (d) Suppose x > y. From (6), x − y > y − y = 0. From exercises 1(h) and 2(c), −(x − y) = −x + y < 0, so
−x + y − y = −x < 0 − y = −y. Conversely, suppose −x < −y. From (2) and (6), −x + x = 0 < −y + x = x − y. From
exercises 1(b) and 2(c), −(x − y) = −x + y < 0, so x − x + y = y < x + 0 = x.
Part (e) If z < 0, then −z > 0 by exercise 2(d). Applying (6) and exercise 1(e), we have x(−z) = −(xz) > y(−z) =
−(yz). From exercise 1(d), xz < yz.
Part (f) If x > 0, then x · x = x2 > 0 · x = 0 by (6). If x < 0, then −x < 0 by exercises 2(c) and (d). Applying (6)
and exercises 1(b), (e), and (f), we have:

(−x)(−x) = −[x · (−x)] = −[−(x · x)] = x2 > 0 · (−x) = 0.

Part (g) Let x ∈ R where x > 0. It follows that x has multiplicative inverse 1/x. From (1) and exercise 1(f), since
x2 > 0, we have:
x2 · (1/x · 1/x) = 1 > 0 · (1/x · 1/x) = 0.
From exercise 2(c), −1 < 0. By the transitivity of the order relation, −1 < 0 < 1.
Part (h) Suppose xy > 0. We can prove the implication by cases. Clearly from exercise 2(b), neither x nor y may be
equal to zero. If x, y > 0, then by (6) and exercise 1(b) x · y > 0 · y = 0. If x, y < 0, then −x, −y > 0 by exercise 2(c).
Applying (6) and exercises 1(b) and (e):

(−x)(−y) = −[x(−y)] = −[−(xy)] = xy > 0 · (−y) = 0.

Now assume that x > 0 and y < 0. From exercise 2(e), we infer from x > 0 that xy < 0 · y = 0, which is a
contradiction. Therefore, it cannot be that x > 0 and y < 0. Having exhausted all possibilities for x and y, we infer that
x and y are either both positive or both negative.
Conversely, suppose x and y are either both positive or both negative. As shown above, in either of those cases xy > 0.
Part (i) Since x > 0, it has multiplicative inverse 1/x by (4). Because x · 1/x = 1 > 0 by exercise 2(g), it follows from
exercise 2(h) that 1/x > 0.
Part (j) Since x, y > 0, there are multiplicative inverses 1/x and 1/y, each of which is greater than zero by exercise
2(i). By exercise 2(h), 1/x · 1/y > 0. Since x > y by hypothesis, by (1) and (2) and exercise (h):
   
1 1 1 1 1 1
x · = >y · = .
x y y x y x

Page 4
Part (k) Since x < y by hypothesis, from (6) and exercise 2(a) we have x + x = 2x < y + x = x + y. Similarly,
y + y = 2y > x + y. From exercises 2(a) and (g), we infer that 1 + 1 = 2 > 0 + 0 = 0. From exercise 2(i), and 2(j), we
have 1/2 > 0. From exercise 2(b) and the transitivity of order relations, we have:
1 1 x+y 1
2x · = x < (x + y) · = < 2y · = y.
2 2 2 2

Problem 4.3. (a) Show that if A is a collection of inductive sets, then the intersection of the elements of A is an inductive set.
(b) Prove the basic properties (1) and (2) of Z>0 .

Solution: Part (a) Let A be the collection of all inductive sets and A = A0 ∈A A0 . Since each A0 ∈ A is an inductive set,
T
1 ∈ A0 , so 1 ∈ A. If x ∈ A, then x ∈ A0 for all A0 ∈ A. Again, since A0 is inductive, x + 1 ∈ A0 . Therefore, x + 1 ∈ A.
Consequently, A is an inductive set.
Part (b) Property (1): Because Z>0 = A0 ∈A A0 where A0 is the collection of all inductive sets of R, from part (a)
T
Z>0 is an inductive set.
Property (2): Suppose A is an inductive set of positive integers. If x ∈ A, then x ∈ Z>0 by the definition of A.
Therefore A ⊆ Z>0 . Conversely, we know 1 ∈ A and if x ∈ A, then x + 1 ∈ A. As a result, 1 + 1 = 2 ∈ A, 2 + 1 = 3 ∈ A,
and so on such that every positive integer is a member of A. It follows that Z>0 ⊆ A and A = Z>0 .

Problem 4.4. (a) Prove by induction that given n ∈ Z>0 , every nonempty subset of {1, . . . , n} has a largest element.
(b) Explain why you cannot conclude from (a) that every nonempty subset of Z>0 has a largest element.

Solution: Note that N = Z>0 (by my definition, where zero is not a natural number), so everything true for one is true of
the other. I will generally use N instead of Z>0 .
Part (a) We will show by induction that for n ∈ N, any nonempty subset Sn+1 = {1, . . . , n} of N has a largest element.
For n = 1, the set S2 = {1} has only one nonempty subset ({1}), which has the largest element 1. Now assume the
inductive hypothesis is true for some n ∈ N. Let S 0 ⊆ Sn+2 = {1, . . . , n + 1}. If n + 1 ∈ S 0 , then n + 1 is the largest
element of S 0 . If n + 1 ∈/ S 0 , then S 0 is some non-empty subset of Sn+1 . By the inductive hypothesis, S 0 has a largest
element. Since this is true for all n ∈ N, the proposition is true for any non-empty subset Sn+1 of N.
Part (b) The result in (a) is limited to bounded subsets of N; it doesn’t hold true for unbounded subsets. Let
Tn = {x ∈ N : x ≥ n} for n ∈ N. Obviously Tn ⊆ N and is non-empty. Suppose Tn has a largest element M . It follows
that M ∈ N. Since N is an inductive set, M + 1 ∈ N. But M + 1 > M > n, so M + 1 ∈ Tn , which contradicts that M is
the largest element of Tn . Therefore Tn does not have a largest element.

Problem 4.9. (a) Show that every nonempty subset of Z that is bounded above has a largest element. (b) If x ∈ / Z, show
there is exactly one n ∈ Z such that n < x < n + 1. (c) If x − y > 1, show there is at least one n ∈ Z such that y < n < x.
(d) If y < x, show there is a rational number z such that y < z < x.

Solution: Part (a) Let A ⊆ Z that is nonempty and bounded above by some M ∈ Z. Let A0 = {M + 1 − a : a ∈ A}.
Note that we can map between corresponding a ∈ A and a0 ∈ A0 as follows: a0 = M + 1 − a and a = M + 1 − a0 .
Since M − a ≥ 0 for all a ∈ A, it follows that M + 1 − a ≥ 1. As a result, a0 ≥ 1 for all a ∈ A, so A0 ⊆ N. From the
Well-Ordering Principle, A0 has a least element m0 . The corresponding element of A is m = M + 1 − m0 .
Simple arithmetic shows since m0 ≤ a0 , we have M + 1 − m ≤ M + 1 − a, so −m ≤ −a. Because this implies that
m ≥ a for all a ∈ A, we conclude that A has a largest element. Since our choice of A was arbitrary, we conclude that any
non-empty subset of Z that is bounded above has a largest element.
Part (b) Let x ∈ R\Z and Sx = {n ∈ Z : n < x}. Since Sx is a non-empty subset of Z (because Z is not bounded
below), it has a greatest element n ∈ Z that is less than x. Because n + 1 ∈/ Sx , we infer that n + 1 ≥ x. It cannot be
that n + 1 = x because x is not an integer, so n + 1 > x.
Now suppose there is an m ∈ Z where m < x < m + 1. It follows that m ∈ Sx , so m ≤ n. If m < n, then
m + 1 ≤ n < x, which is not possible. Therefore m = n, from which it follows that n is unique.
Part (c) Let x, y ∈ R where x − y > 1. From part (b), there is an n ∈ Z such that n < x < n + 1. It follows that
x − 1 < n. Because y < x − 1, we conclude that y < n < x for some n ∈ Z.
Part (d) Let x, y ∈ R where y < x. If x − y > 1, by part (c) there is an n ∈ Z such that y < n < x. Since n is a
rational number as well, we’re done.

Page 5
If 0 < x − y ≤ 1, we will show that there exists an m ∈ Z such that m(x − y) > 1. Let a = x − y, so 0 < a ≤ 1. It
follows that 2/a ≥ 2 > 1. From part (b), there is an m ∈ Z such that m − 1 < 2/a < m. Therefore:
2 2
2≤ · a = (x − y) < m(x − y) = mx − my.
a a
Therefore, mx − my > 1. From part (c), there is an n ∈ Z such that my < n < mx. Since m > 2/a > 1, we know
m 6= 0. Therefore, y < n/m < x, where n/m is a quotient of two integers and is therefore a rational number.

Problem 4.11. Given m ∈ Z, we say that m is even if m/2 ∈ Z and m is odd otherwise. (a) Show that if m is odd,
m = 2n + 1 for some n ∈ Z. (b) Show that if p and q are odd, so are p · q and pn , for any n √
∈ Z>0 . (c) Show that if a > 0 is
rational, then a = m/n for some m, n ∈ Z where not both m and n are even. (d) Theorem: 2 is irrational.

Solution: Part (a) Suppose m is odd so m/2 ∈ / Z. Consequently, m ∈ R\Z, so by exercise 9(b) there is an n ∈ Z such
that n < m/2 < n + 1. We then have 2n < m < 2n + 2. Since 2n + 1 is the only integer between 2n and 2n + 2, it
follows that m = 2n + 1.
Part (b) If p, q ∈ Z are odd, then from part (a) there are m, n ∈ Z such that p = 2m + 1 and q = 2n + 1. Therefore:

p · q = (2m + 1)(2n + 1) = 4mn + 2m + 2n + 1 = 2(2mn + m + n) + 1 = 2r + 1,

where r ∈ Z and r = 2mn + m + n. We conclude that p · q is odd.


We will prove that pn is odd for n ∈ N by induction. Clearly p1 = p, which is odd. If the inductive hypothesis is true
for some n ∈ N, then pn+1 = pn · p, which is the product of two odd numbers by the inductive hypothesis. By the previous
result, pn+1 is odd. We conclude that pn is odd for all n ∈ N.
Part (c) Let a ∈ Q be given where a > 0. Clearly a may be expressed as the quotient of two natural numbers. Let
Ba = {x ∈ N : x · a ∈ N}. By the Well-Ordering Principle, Ba has a smallest element n, and n · a = m for some m ∈ N.
It follows that a = m/n, the quotient of two natural numbers. Now suppose both m and n are even, so there are r, s ∈ N
such that m = 2r and n = 2s. Then a = 2r/2s = r/s and s · a = r. Therefore s ∈ Ba , but s < n, which contradicts that
n is the smallest element
√ of Ba . We conclude that m and n cannot both be even. √
Part (d) Assume 2 is rational. From part (c), there are m, n ∈ Z where m/n = 2 and m and n are not both even.
We then have m2 /n2 = 2, so m2√= 2n2 . We infer that m2 is even, so by part (b) m is even. Therefore there is an r ∈ Z
such that m = 2r and 2r/n = 2. Squaring the previous expression, we have 4r2 = 2n2 , so n2 = 2r2 . Following the
previous
√ argument, we conclude that n is even as well, which contradicts that m and n are not both even. It follows that
2 is irrational.

Section 5
Problem 5.1. Show there is a bijective correspondence of A × B with B × A.

Solution: Let f : A × B → B × A where f (x, y) = (y, x). If f (x0 , y0 ) = f (x1 , y1 ), then (y0 , x0 ) = (y1 , x1 ), implying that
(x0 , y0 ) = (x1 , y1 ). Therefore f is injective. For (y, x) ∈ B × A, it follows that f (x, y) = (y, x) where (x, y) ∈ A × B.
Therefore f is surjective. It follows that f is bijective.

Problem 5.3. Let A = A1 × A2 · · · and B = B1 × B2 × · · · . (a) Show that if Bi ⊂ Ai for all i, then B ⊂ A. (b) Show the
converse of (a) holds if B is nonempty. (c) Show that if A is nonempty, each Ai is nonempty. Does the converse hold? (We
will return to this question in the exercises of §19.) (d) What is the relation between the set A ∪ B and the cartesian product
of the sets Ai ∪ Bi ? What is the relation between the set A ∩ B and the cartesian product of the sets Ai ∩ Bi ?

S
Solution: Part (a) Suppose Bi ⊆ Ai . Let X = i∈N Bi and x : N → X so it is an m-tuple where x ∈ B. Because each
Bi is a subset of Ai , it follows that if x(i) ∈ Bi , then x ∈ Ai (ignoring, of course, the codomain mismatch as raised in the
text). Since this is true for every i, we infer that x ∈ A, so B ⊆ A.
Part (b) Suppose B is non-empty and B ⊆ A. Let x ∈ B, from which it follows that x ∈ A. For each valid i, if
bi ∈ Bi , then there is some y ∈ B such that y(i) = bi . Since y ∈ A, it follows that y(i) ∈ Ai . As this is true for all
elements of Bi , it follows that Bi ⊆ Ai .

Page 6
S
Part (c) If A is nonempty, there is some y ∈ A where y : N → Y where Y = i∈N Ai . For k ∈ N, we have y(k) ∈ Ak ,
so Ak is nonempty.
The converse implicitly requires the axiom of choice. If each Ak is nonempty, let ak be an arbitrary element of Ak for
k ∈ N. It follows that (a1 , a2 , . . .) ∈ A, so A is nonempty.
Part (d) The set A ∪ B is the set of all ω-tuples contained in A or B. Let C = (A1 ∪ B1 ) × (A2 × B2 ) × · · · , which
contains all ω-tuples x where x(k) ∈ Ak ∪ Bk . If y ∈ A ∪ B, then y(k) ∈ Ak or y(k) ∪ Bk . Therefore y(k) ∈ Ak ∪ Bk .
Since this is true for all k, we conclude that y ∈ C, so A ∪ B ⊆ C. Note that C is not a subset of A ∪ B because x ∈ C
may mix elements of A and B for different k.
The set A ∩ B is the set of all ω-tuples contained in both A and B. It follows that if y ∈ A ∩ B, then y(k) ∈ Ak ∩ Bk
for all k ∈ N. Let D = (A1 ∩ B1 ) × (A2 ∩ B2 ) × · · · . If x ∈ D, then x(k) ∈ Ak ∩ Bk , from which we conclude that
x ∈ A ∩ B. By the same argument, each y ∈ A ∩ B is also an element of D. Therefore, A ∩ B = D.

Problem 5.5. Which of the following subsets of Rω can be expressed as the cartesian product of subsets of R?
(a) {x : xi is an integer for all i}.
(b) {x : xi ≥ i for all i}.
(c) {x : xi is an integer for all i ≥ 100}.
(d) {x : x2 = x3 }.

Solution: I refer to the sets defined in questions (a) through (d) as sets A, B, C, and D, respectively.
Part (a) Yes. The set Z is a subset of R, and Zω is obviously the cartesian product that generates A.
Part (b) Yes. Let Si = {z ∈ R : z ≥ i} and x ∈ S1 × S2 × · · · . It follows that x(k) ≥ k for each k ∈ N, so x ∈ B.
Now let y ∈ B. For k ∈ N, we have y(k) ≥ k, so this m-tuple is an element of S1 × S2 × · · · . The two sets are therefore
equal.
Part (c) Yes. For k ∈ N, let Sk be defined as followed:

R for 1 ≤ k ≤ 99,
Sk =
Z for k ≥ 100.

If x ∈ S1 × S2 × · · · , then x(k) ∈ R for 1 ≤ k ≤ 99 and x(k) ∈ Z for k ≥ 100. The values of y are irrelevant for
other k, so it follows that x ∈ C. Now let y ∈ C. By the set’s definition, y(k) ∈ Z for k ≥ 100. Since the set does not
impose any restrictions on other values of k, we infer that y(k) ∈ R for 1 ≤ k ≤ 99. Therefore y ∈ S1 × S2 × · · · and
C = S1 × S2 × · · · .
Part (d) No. Let T = T1 × T2 × · · · where Tk ⊆ R, and suppose T = D. If x ∈ D, then x(2) ∈ R because its value
is unrestricted other than equaling x(3). We infer that A2 = R. Because x(3) = x(2), it must also be that A3 = R.
Accordingly, T = T1 × R × R × T4 × · · · . Clearly then, there is a (t1 , t2 , t3 , t4 , . . .) ∈ T where t2 6= t3 , so x ∈
/ T, a
contradiction. Therefore, there is no cartesian product of subsets of real numbers that equals D.

Section 6
Problem 6.2. Show that if B is not finite and B ⊂ A, then A is not finite.

Solution: If A is finite, there is a bijection f : A → Sn . Since B ⊆ A, the restriction of f to B is an injection onto a


section of natural numbers, so B is finite by Corollary 6.7. Because this is a contradiction, we conclude that A is not finite.

Problem 6.3. Let X be the two-element set {0, 1}. Find a bijective correspondence between X ω and a proper subset of itself.

Solution: Let Y = {(0, x1 , x2 , . . .) : (x1 , x2 , . . .) ∈ X ω }. We will show that Y is a proper subset of X ω . Let y ∈ Y . For
each k ∈ N, we have y(k) ∈ X; therefore, y ∈ X ω , so Y ⊆ X ω . We see, however, that (1, x2 , x3 , . . .) is contained in X ω
but not in Y . It follows that Y is a proper subset of X ω .
Let g : X ω → Y where f (x1 , x2 , . . .) = (0, x1 , x2 , . . .). Clearly, if (x1 , x2 , . . .) 6= (x01 , x02 , . . .), then f (x1 , x2 , . . .) 6=
f (x01 , x02 , . . .). Further, if z ∈ Y where z = (0, x1 , x2 , . . .), then (x1 , x2 , . . .) ∈ X ω and f (x1 , x2 , . . .) = z. Therefore g is a
bijection of X ω onto a proper subset of itself. Note that from Corollary 6.3, X ω is not finite.

Page 7
Problem 6.4. Let A be a nonempty finite simply ordered set. (a) Show that A has a largest element. (b) Show that A has
the order type of a section of positive integers.

Solution: Part (a) As suggested by the hint, we will show that if A has cardinality n ∈ N, then it has a largest element.
For n = 1, the set A has a single element, which must be its largest element. If we assume the inductive hypothesis
holds for n ∈ N, then A = An ∪ A1 where A has cardinality n and A1 has cardinality 1. By the inductive hypothesis, An
has largest element m and A’s single element p is its largest element. The largest element of A is sup{m, p}. Since this
proposition holds for all n ∈ N, we conclude that every nonempty finite set with an order relation has a largest element.
Part (b) Since A is finite, there is a bijection f : A → {1, . . . , n} where n is A’s cardinality. Let g : {1, . . . , n} →
{1, . . . , n}. We will define g inductively. Let An = A and let mn be the largest element of An (which must exist per
part (a)). Let g(f (mn )) = n. Not let An−1 = An \mn , which has largest element mn−1 . Let g(f (mn−1 )) = n − 1. We
continue until we reach A0 = ∅ and we’re done. Clearly g is a bijection. Let h : A → {1, . . . , n} where h = g ◦ f , which is
a bijection by exercise 3.4.
Now suppose a, b ∈ A where a < b. It follows that mp = a and mq = b for some p, q ∈ N. Since a < b, it must be
that p < q, so h(a) = p < q = h(b). Therefore A is of the same order type as a section of natural numbers.

Problem 6.5. If A × B is finite, does it follows that A and B are finite?

Solution: Assuming both A and B are nonempty, yes. If A × B is finite, there is a bijection f : A × B → Sn for some
n ∈ N. Let g : A → Sn . For each a ∈ A, let αa = {m ∈ N : m = f (a, b) for some b ∈ B}. By the Well-Ordering Principle,
αa has a least element ma , and we can let g(a) = ma .
We will show that g is injective. Because f is a bijection, if a1 , a2 ∈ A and a1 6= a2 , then f (a1 , b1 ) 6= f (a2 , b2 ) for any
b1 , b2 ∈ B. It follows that each the set of αa are pairwise disjoint. Accordingly, g(a1 ) 6= g(a2 ), so g is an injection of A
into a section of natural numbers. By Corollary 6.7, A is finite.
Following the same argument, B is finite as well.
(There is a simpler solution (which I didn’t come up with) where one fixes a b ∈ B then uses the restriction of f
to A × {b}, which is injective into a section of natural numbers. You can compose this restriction with a bijection of
A → A × {b}.
Note that if either A or B or empty, then A × B = ∅ (which is finite), but the other set could be infinite.

Problem 6.6. (a) Let A = {1, . . . , n}. Show there is a bijection of P(A) with the cartesian product X n , where X is the
two-element set X = {0, 1}. (b) Show that if A is finite, then P(A) is finite.

Solution: Part (a) We will construct f : P(A) → X n as follows. For p ∈ P(A) define xp ∈ X n as follows: If k ∈ p,
then xp = 1; otherwise, xp = 0. In computer science terms, xp is a bitset indicating by the k’th bit whether k is contained
in p.
We will show that f is bijective. Suppose f (p1 ) = f (p2 ) = yp for some p1 , p2 ∈ P(A). It follows that for 1 ≤ k ≤ n,
if k ∈ p1 , then yp (k) = 1, so k ∈ p2 , and p1 ⊆ p2 . By the same argument, p2 ⊆ p1 , so p1 = p2 . Now let xq ∈ X n . Let
q = {k ∈ {1, . . . , n} : xq (k) = 1. Clearly q ∈ P(A) and f (q) = (x)q . We conclude that f is bijective.
Part (b) By Corollary 6.8, X n is finite because it a finite cartesian product of identical finite sets X. Therefore there
is a bijection g : X n → Sm for some m ∈ N. From part (a), since A is finite there is a bijection f : P(A) → X n . If
h = g ◦ f , then h is a bijection from P(A) to Sm . We conclude that P(A) is finite by Corollary 6.7.

Problem 6.7. If A and B are finite, show that the set of all functions f : A → B is finite.

Solution: Suppose A and B are finite. Let F = {f : A → B}. If f ∈ F , it has associated with it a rule of assignment
rf ⊆ A × B, so rf ∈ P(A × B). Let R = {rf : f ∈ F }. It follows that R ⊆ P(A × B). As we showed in exercise
6(b), P(A × B) is finite because A and B are finite. Applying Corollary 6.6, R is finite. Therefore there is a bijection
g : R → {1, . . . , n} for some n ∈ N.
Note that for f1 , f2 ∈ F , if f1 6= f2 , each has a distinct rule of assignment; otherwise, they would be the same function.
We can therefore define the injective function h : F → R where h(f ) = rf . If j : F → {1, . . . , n} where h = g ◦ h, then j
is an injection from F to a section of natural numbers because both g and h are injective (see exercise 2.4). From Corollary
6.7, F is finite.

Page 8
Section 7
Problem 7.1. Show that Q is countably infinite.

Solution: Define g : N × N → Q≥0 where g(m, n) = (m − 1)/n. If q ∈ Q≥0 , then q = m0 /n0 for some m0 ∈ Z≥0 and
n ∈ N. If m00 = m0 + 1, then m00 ∈ N and q = (m00 − 1)/n0 ; it follows that g(m00 , n0 ) = q and g is surjective. Since N × N
is countable, there is a surjection f : N → N × N, so g ◦ f is a surjection from N → Q≥0 . From Theorem 7.1, Q≥0 is
countable. We conclude that Q≥0 is countably infinite because it contains the positive integers.
Following a very similar argument, we can show that Q<0 is countably infinite using the surjection h : N × N → Q<0
where h(m, n) = −m/n.
Since Q = Q≥0 ∪ Q<0 , each of which is countably infinite, we infer from Theorem 7.5 that Q is countably infinite.

Problem 7.3. Let X be the two-element set {0, 1}. Show there is a bijective correspondence between the set P(Z+ ) and the
cartesian product X ω .

Solution: This is nearly identical to exercise 6.6(a). Again, we treat X ω as a bitset where the k’th bit indicates whether
k ∈ N is a member of a given element of P(N). Let f : P(N) → X ω . If p ∈ P(X), then f (p) = xp where xp (k) = 1 if
k ∈ p and xp (k) = 0 otherwise. As shown in exercise 6.6(a), this function is a bijection.

Problem 7.4. (a) . . . Assuming each polynomial equation has only finitely many roots, show that the set of algebraic numbers
is countable. (b) . . . Assuming the reals are uncountable, show that the transcendental numbers are uncountable.

Solution: Part (a) Let Fn be the set of all polynomials of degree n with rational coefficients. We may form a map
fn : Fn → Qn where if f (x) = an tn + . . . a0 , then fn (f ) = (an , . . . , a0 ). It is easily shown that fn is a bijection. From
Theorem 7.7, Qn is countable, so there is a bijection gn : Qn → N, from which it follows that hn = gn ◦ fn is a bijection
from Fn to N. The set Fn is therefore countable.
Let An be the set of all algebraic numbers that are a non-zero root of some polynomial in Fn . Let bf = {a ∈ A :
f (a) = 0}. We know from algebra that a polynomial of degree n has at most n roots. Let Xn = Fn × {1, . . . , n}, which
is countable by Theorem 7.7. Therefore there is a bijection ln : Xn → N. Now let r : Xn → An , which we define as a
mapping of the k’th root of some polynomial of degree n to an algebraic number. If (f, k) ∈ Xn , let r(f, k) = sf (k) be a
tuple of the non-zero roots of f , defined inductively as follows.

sf (1) = 0 if bf = ∅; otherwise, inf bf ,
sf (k) = 0 if bf \s({1, . . . , k − 1}) = ∅; otherwise, inf(bf \s({1, . . . , k − 1}); k > 1.
Since each bf is finite, it follows that it (and all non-empty subsets of it) have infima. Therefore sf is well-defined.
We will show that r is surjective. Let a ∈ An . By definition, there is some f ∈ Fn where f (a) = 0, so a ∈ bf .
Therefore a ∈ r(f, {1, . . . , n}).
Now we will show that An is countable. From the prior results, we know there are surjections l−1 : N → Xn and
r : Xn → An . Their composition t = r ◦ l−1 is a surjection from N → An . From Theorem 7.1, An is countable.
The set of all algebraic numbers is the union of the sets of algebraic numbers for the polynomials of n ∈ N. Note that
polynomials of degree 0 are constant and therefore have no roots except in the trivial case of f (x) = 0. Observe further
that above we only accounted above for non-zero roots; we know, however, that 0 is an algebraic number because it is the
root in the polynomial f (x) = x. Therefore, the set A of all algebraic numbers is:

[
A = {0} ∪ An
k=1

Because A is a countable union of countable sets, we conclude from Theorem 7.5 that the set of algebraic numbers is
countable. Note that Q is a subset of A because if q ∈ Q, then q is a root of f (x) = x − q. It follows that A is countably
infinite.
Part (b) Suppose the set T of all transcendental numbers is countable. Since R = A ∪ T , we conclude from Theorem
7.5 that the real numbers are countable, which is a contradiction. Thus, T is uncountable.

Problem 7.5. [See question.]

Page 9
Solution:
Part (d) Applying the diagonalization technique used in the proof of Theorem 7.7, we can show that D is uncountable.
Let f : N → D where f (n) = (xn1 , xn2 , . . . , xnm , . . .). Let y ∈ D where y(n) = 2xnn . It follows that f (n) 6= y for any
n ∈ N because y(n) 6= xnn . Therefore f is not surjective. Since this is true for any arbitrary function from N → D, we
conclude by the contrapositive of Theorem 7.1 that D is not countable.
Part (e) Following the same argument in part (d), E is uncountable.
Part (f) In all cases here, the functions are from N → {0, 1}. For n ∈ N, let Fn be the set of functions f such that
f (k) = 0 for k ≥ n. Let F1 = {f1 (x) = 0}, thus containing the only function in F that is eventually zero at N = 1. For
n > 1, let gn : Fn → Nn−1 . If f ∈ Fn , then let g(f ) = xf ∈ Nn−1 where x(k) = f (k) for 1 ≤ k < n. We will show that gn
is injective. Suppose f1 , f2 ∈ Fn and g(f1 ) = g(f2 ) = x1 . It follows that for 1 ≤ k < n, we have f1 (k) = x1 (k) = f2 (k),
and for f ≥ 0 we have f1 (k) = f2 (k) = 0. Accordingly, f1 = f2 .
Since Nn−1 is countable by Theorem 7.7, there is an injection hn : Nn−1 → N. As a result, the composition hn ◦ gn :
Fn → N is injective. Applying Theorem 7.1, infer that Fn is countable. S

The set F of all functions N → {0, 1} that are eventually zero is F = k=1 Fk . Since each Fk in this countable union
is countable, it follows from Theorem 7.5 that F is countable.
Part (g) Following the same argument in part (f), we see that G is countable.
Part (h) Following the same argument in part (f), we see that H is countable.

Problem 7.7. Show that the sets D and E in Exercise 5 have the same cardinality.

Solution: Let g : E → D where if fe ∈ E, then g(fe ) = fe . Since {0, 1} ⊂ N, clearly fe ∈ E. Now we show that g is
injective. Let fe1 , fe2 ∈ E for some g(fe1 ) = g(fe2 ). It follows that fe1 = fe2 .
Let h : D → E. This one is a little trickier. If fd ∈ D, let h(fd ) = fe , which we define as follows. For each n ∈ N, let
fd (2n 3fd (n) ) = 1. Then set all other fd (k) = 0. We know that if m = 2x 3y (its prime factorization), x and y are uniquely
determined. We can also prove this proposition here just as it was in Corollary 7.4. Let m1 = m2 for some m1 , m2 ∈ N,
and suppose m1 = 2x1 3y1 and m2 = 2x2 3y2 . If x1 < x2 , then 3y1 = 2x2 −x1 3y2 , which contradicts that 3 is odd. Therefore
x1 = x2 . Therefore 3y1 = 3y2 , from which we infer that y1 = y2 . By the contrapositive, if x1 6= x2 or y1 6= y2 , then
m1 6= m2 . Applying this here, since each (x, y) ∈ fd is unique (since every element in the domain appears only once), we
are assured that each fd (k) = 1 corresponds to a element of fd .
We will now show that h is injective. Suppose there are fd1 , fd2 ∈ D where h(fd1 ) = h(fd2 ) = fe It follows
that if fe (m) = 1, then m = 2x 3y for some x, y ∈ N. As shown above, x and y are uniquely determined by m, so
fd1 (x) = y = fd2 (x). Any k where fe (k) = 0 may be ignored because those k do not define either function. Since this is
true for all n ∈ N whose image under fe is one, we conclude that fd1 = fd2 .
Having found injections from D → E and E → D, we infer from exercise 6(b) that D and E have the same cardinality.

Section 8
Problem 8.4. The Fibonnaci numbers of number theory are defined recursively by the formula:

λ1 = λ2 = 1,
λn = λn−1 + λn−2 for n > 2.
Define them rigorously by use of Theorem 8.4.

Solution: Define ρ as follows:



1 for n = 1,
ρ(f |{1, . . . , n}) =
f (n) + f (n − 1) for n ≥ 2.
Let a0 = 1, in which case:

h(1) = a0 ,
h(k) = ρ(h|{1, . . . , k − 1}) for k ≥ 2.
We then have h(1) = 1, and h(2) = ρ(f |{1}) = 1, and h(k) = ρ(f |{1, . . . , k − 1})) = h(k − 1) + h(k − 2) for k ≥ 3,
thus satisfying the definition of the Fibonacci sequence.

Page 10
Problem 8.5. Show that there is a unique function h : Z+ → R+ satisfying the formula:

h(1) = 3,
h(i) = [h(i − 1) + 1]1/2 for i > 1.

Solution: p
Let ρ(f |{1, . . . , n}) = f (n) + 1 for n ∈ N and a0 = 3. We then have:

h(1) = a0 ,
h(k) = ρ(h|{1, . . . , k − 1}) for k > 1.
We must show that ρ(h) is well-defined. We will show p by induction that ρ(h|{1, . . . , n}) has a unique value greater
than zero for each n ∈ N. If n = 1, then ρ(h|{1}) = h(1) + 1 = 2 > 0. If we assume the inductive hypothesis is true
for some n ∈ N, then:
p p
ρ(h|{1, . . . , n + 1}) = h(n + 1) + 1 = ρ(h|{1, . . . , n}) + 1.
p
By the inductive hypothesis, ρ(h|{1, . . . , n}) > 0 and is well-defined, so ρ(h|{1, . . . , n + 1}) > 0 and is well-defined.
Our claim is thus valid for all n ∈ N. Under the Principle of Recursive Definition (Theorem 8.4), h uniquely satisfies the
formula at issue.

Problem 8.6. [See question.]

√ √
qp
Solution: Part (a) We see that h(4) = 2 − 1 = 1 ≈ −0.586, which has no real solution. Therefore h is not
well-defined. This formula does not violate the Principle of Recursive Definition because there is no well-defined ρ, so the
Principle does not apply.
Part (b) Let ρ(f : {1, . . . , n} → R>0 ) be defined as follows:

5 for f (m) ≤ 1,
ρ(f |{1, . . . , m}) = p
f (m) − 1 for f (m) > 1.
Letting a0 = 3, we have:

h(1) = a0 ,
h(k) = ρ(h|{1, . . . , k − 1}) for k > 1.
As in exercise 8.5, we can show by induction that ρ is well-defined. By the Principle of Recursive Definition, h uniquely
satisfies the given recursion formula.

Problem 8.8. Verify the following version of the principle of recursive definition: Let A be a set. Let ρ be a function assigning,
to every function f mapping a section Sn of Z+ into A, an element ρ(f ) of A . Then there is a unique function h : Z+ → A
such that h(n) = ρ(h|Sn ) for each n ∈ Z+ .

Solution: The only difference between the two formulations is that Theorem 8.4 does not use ρ in the definition of h(1),
whereas the formulation in the prompt requires h to depend on ρ for all n ∈ N. We’ll see that the two versions are
equivalent.
Let ρ1 , a0 , and h1 be as defined by Theorem 8.4 for a valid recurrence. Define a function ρ2 : N → A as follows:

a0 for m = 1,
ρ2 (f |{1, . . . , m}) =
ρ1 (f |{1, . . . , m − 1}) for m ≥ 2.
As stated in the prompt, let h2 (k) = ρ2 (h2 |{1, . . . , k}) for all k ∈ N. It follows that h2 (1) = ρ2 (h2 |{1}) = a0 = h1 (1).
If k ≥ 2 and h1 |{1, . . . , k − 1} = h2 |{1, . . . , k − 1}, we have h2 (k) = ρ2 (h2 |{1, . . . , k}) = ρ1 (h2 |{1, . . . , k − 1}) =
ρ1 (h1 |{1, . . . , k − 1}) = h1 (k). Therefore h1 = h2 for all k ∈ N.
Now let ρ2 and h2 be as defined by the principle in the prompt for a valid recurrence. Let a0 = ρ2 (f |{1}). Define
ρ1 : N → A as ρ1 (f |{1, . . . , m}) = ρ2 (f |{1, . . . , m + 1}). Let h1 be as it is defined in Theorem 8.4. It follows that
h1 (1) = a0 = h2 (1). If k ≥ 2 and h1 |{1, . . . , k − 1} = h2 |{1, . . . , k − 1}, then h1 (k) = ρ1 (h1 |{1, . . . , k − 1}) =
ρ1 (h2 |{1, . . . , k − 1}) = ρ2 (h2 |{1, . . . , k} = h2 (k). Therefore h1 = h2 . We conclude that the definitions are equivalent.

Page 11
Section 9
Problem 9.1. Define an injective map f : Z+ → X, where X is the two-element set {0, 1}, without using the choice axiom.

P∞
Solution: For x ∈ N, map x to its binary representation. We can uniquely express x as k=1 2k−1 ak where (ak ) is a
sequence over {0, 1}. Therefore let f (x) = (a1 , a2 , . . .). Since this representation is unique, f is injective. Note that the
axiom of choice is not necessary.

Problem 9.5. (a) Use the choice axiom to show that if f : A → B is surjective, then f has a right inverse h : B → A.
(b) Show that if f : A → B is injective and A is nonempty, then f has a left inverse. Is the axiom of choice needed?

Solution: Part (a) Let h be defined as follows. For each b ∈ B, let Cb = {a ∈ A : f (a) = b}, and let C be the collection
of all Cb . Since f is surjective, no Cb is empty. Moreover, the fact that f is a function ensures that the sets contained in C
are pairwise disjoint. By Lemma 9.2, there is a choice function φ : C → A such that if Cb ∈ C, then f (Cb ) is some element
of Cb ⊆ A. For each b ∈ B, let h(b) = φ(Cb ). Hence we have associated a single element of A for each element of B.
If b ∈ B, there is an a ∈ Cb such that f (b) = a. It follows that h(a) = b, so f ◦ h = ib for all b ∈ B. Consequently, f
has a right inverse.
Part (b) Let g : B → A be defined as follows. For b ∈ f (A), let g(b) = a where f (a) = b. Since f is injective, there
is a unique a associated with each b; therefore, the foregoing is well-defined. Now let a0 ∈ A be arbitrary. (This does not
require the axiom of choice because we’re only making a single choice as opposed to infinite number of arbitrary choices.)
For each b ∈ B\f (A), let g(b) = a0 . Thus g is defined for all over B.
If a ∈ A, then f (a) = b ∈ F (A), so g(b) = a. Consequently, g ◦ f = ia for all a ∈ A, so f has a left inverse. Moreover,
the axiom of choice is not needed.

Section 10
Problem 10.1. Show that every well-ordered set has the least upper bound property.

Solution: Let S be a well-ordered set and let T be a non-empty subset of it that is bounded above by some u ∈ S. Let
U be the set of eall upper bounds of T in S, which must contain at least u. It follows that U has a smallest element v.
Consequently, sup T = v. Since this is true for every non-empty subset of S that is bounded above, S has the least upper
bound property.

Problem 10.2. (a) Show that in a well-ordered set, every element except the largest (if one exists) has an immediate successor.
(b) Find a set in which every element has an immediate successor that is not well-ordered.

Solution: Part (a) Suppose the set S is well-ordered. Let a, b ∈ S where a < b. By definition, (a, b) = {c ∈ S : a < c < s}.
If (a, b) is empty, b is the immediate successor of a. Otherwise, (a, b) has a smallest element γ, from which it follows that
(a, γ) is empty (otherwise, γ wouldn’t be the smallest element of (a, b)). Therefore γ is the immediate successor of a. We
conclude that every element of S that is not the largest element (if there is one) has an immediate successor.
Part (b) The set of integers with the ordinary order relation.

Problem 10.4. (a) Let Z− denote the negative integers in the usual order. Show that a simply ordered set A fails to be
well-ordered if and only if it contains a subset having the same order types as Z− .
(b) Show that if A is simply ordered and every countable subset of A is well-ordered, then A is well-ordered.

Solution: Part (a) Suppose there is a subset A0 of A that has the same order type as Z<0 , in which case there is a
bijection f : A0 → Z<0 such that if a, b ∈ A0 and a < b, then f (a) < f (b). Let X be a subset of Z<0 that does not have
a smallest element (which must exist because Z<0 is not well-ordered). If we let A1 = f −1 (X), then A1 ⊆ A0 ⊆ A. Let
α ∈ A1 . It follows that there is some x ∈ X such that x < f (α) since X has not smallest element. Therefore there is a
β = f −1 (x) such that β < α. Accordingly, A1 has no smallest element. By definition A is not well-ordered.
Conversely, suppose A is not well-ordered. For each a ∈ A, let Ca = {a0 ∈ A : a0 < a}, and let C be the collection of all
Ca and A. It follows that there is a choice function c : C → A. Let f : Z<0 → A be defined as follows. Let f (−1) = c(A),

Page 12
then for k ≥ 2, let f (−k) = c(Cf (−k+1) ). It is easy to show that f is injective. Suppose j, k ∈ Z<0 where j < k. It follows
that f (j) ∈ Cf (j+1) and for all a ∈ Cf (j+1 ), we have a < f (l) where k < l. Since k < j, we infer that f (j) < f (k). If
g : Z<0 → f (Z<0 ), then g is bijective and, as shown previously, if k < j, then g(k) < g(j). Consequently, there is a subset
f (Z<0 ) of A with the same order type as Z<0 .
Part (b) If A is not well-ordered, from part (a) it must have a subset A0 with the same order type as Z<0 . Since
this requires a bijection f from A0 → Z<0 , it follows that A0 is countable (simply define a bijection g : Z<0 → N). By
hypothesis, every countable subset of A0 is well-ordered, so there can be no such bijection f . We conclude from part (a)
that A is well-ordered.

Problem 10.9. [See question.]

Solution: Part (a) It took me a while to get this order relation straight in my head. I’ll spend a little extra time and care
working through my arguments.
For any n ∈ N, let Sn = {a ∈ A : ak = 1 for k ≥ 2}. In other words if a ∈ Sn , then a = (a1 , . . . , an , 1, 1, 1, . . .).
We will show that Sn is a section of A. Clearly Sn ⊂ Sn+1 . By the definition of the order relation, if a ∈ A, then
a < (1, . . . , 1, 2, 1, 1, . . .) = b where the 2 is at the n + 1 position. This is clear because an+1 = 1 < 2 = bn+1 and
ak = bk = 1 for k ≥ n + 1. On the other hand, if c < b, it must be that ck = 1 for k ≥ n + 1 (since the only natural
number less than the 2 at the n + 1 position of b is 1). It follows that c ∈ Sn . We conclude that an equivalent definition
of Sn is {a ∈ A : a < b}, so (Sn ) is an increasing sequence of sections of A. As a consequence, any element of Sk is less
than any element of Sn \Sk where n > k.
We will now prove that for any n ∈ A, the section Sn has the same order type as Nn . For S1 , we can trivially generate
the bijection f1 : S1 → N1 where if a ∈ S1 , then f (a) = a1 . Obviously the two sets have the same order type. For
n ≥ 2, let d ∈ Sn . Define fn : Sn → Nn where f (d) = (dn , . . . , d). It is clear that fn is injective. Moreover, for any
(x1 , . . . , xn ) ∈ Nn , we have f (xn , . . . , x1 ) = (x1 , . . . , xn ). Therefore fn is a bijection.
Now we will show that fn is order-preserving. Let g, h ∈ Sn and suppose g < h. It follows that there is some j
where gj < hj and gk = hk for j < k ≤ n. By the definition of dictionary ordering, fn (g) = (gn , . . . , gj , . . . , g1 ) <
(hn , . . . , hj , . . . , h1 ) = fn (h). Accordingly, each fn is order-preserving.
We conclude that for every n ∈ N, there is a section of A with the same order type as Nn .
Part (b) Let A0 be a non-empty subset of A and J = {n ∈ N : Sn ∩ A0 6= ∅}. Since A is nonempty and the infinite
union of Sn equals A, we know J cannot be empty. Consequently, J has a least element j. Since the elements of Sj are
smaller than any element in Sn \Sj for n > j, it follows that the smallest element of Sj is the smallest element of A0 .
Therefore A is well-ordered.

Problem 10.11. Let A and B be two sets. Using the well-ordering theorem, prove that either they have the same cardinality,
or one has cardinality greater than the other. [Hint: If there is no surjection f : A → B, apply the preceding exercise.]

Solution: We’ll start with the simple case. If there are injections A → B and B → A then by the Schroeder-Bernstein
Theorem (exercise 7.7(b)), A and B have the same cardinality.
On the other hand, suppose without loss of generality that there is no injection f : B → A. If we assume there is a
surjection g : A → B, we run into the contradiction that we can generate an injection f . By the Well-Ordering Theorem,
there are order relations <A and <B such that A and B are well-ordered. For each b ∈ B, let f (b) = a0 where a0 is the
smallest element of g −1 ({b}), which must be non-empty because g is surjective. We then have our impossible injection f .
From the result in exercise 10.10(a), there is h : A → B where h(a) equals the smallest element of C\h(Sa ). This
function is injective. Let a1 , a2 ∈ A where a1 <A a2 . It follows that h(a1 ) ∈ C\h(Sa1 ) and h(a2 ) ∈ C\h(Sa2 ). Clearly
Sa1 is a subset of Sa2 , so h(a2 ) ∈
/ C\h(Sa1 ). Since h(a1 ) 6= h(a2 ), the function h is injective.
Because there is an injection A → B but no injection B → A, the set B has a greater cardinality than A.

Section 11
Problem 11.1. If a and b are real numbers, define a ≺ b if b − a is positive and rational. Show this is a strict partial order on
R. What are the maximal simply ordered subsets?

Solution: Let a, b, c ∈ R. If b − a is irrational, then a and b are incomparable, so this cannot be a simple ordering on R.
Now suppose b − a is rational. Non-reflexivity is met because if a = b, then b − a is not positive. Suppose b − a ∈ Q>0 ,

Page 13
in which case a ≺ b. If c − b ∈ Q>0 , then b ≺ c. It follows that c − a > b − a > 0 and is rational, so a ≺ c is met.
Consequently, transitivity is met, and this is a strict partial ordering.
The collection of maximal simply-ordered subsets of R consists of sets Ba = {a+b : b ∈ Q} where a ∈ R. Let x, y ∈ Aa
where x 6= y. It follows that x − y = (a + bx ) − (a + by ) = bx − by ∈ Q. This difference is either positive or negative, so
x ≺ y or y ≺ x for all distinct x and y in Ba . Every set Ba is simply-ordered. Now we will show Ba is maximal. If Ba is
a proper subset of some C ⊆ Q, then if z ∈ C\Ba , then z is irrational and cannot be expressed as a + bz for any bz ∈ R.
Accordingly z − a is irrational. Consequently z − y = z − a − by , which must be irrational, so y and z are incomparable;
therefore D is not simply-ordered. It follows that each Ba is maximal.
Any other viable candidate for a maximal simply-ordered subset C 00 of R must include some a pair of real numbers a
and b such that b − a is irrational. Otherwise, for every c, d ∈ C 00 where c 6= d, either the pair is rational (in which case
C 00 ⊆ B0 , which is not maximal if it is a proper subset) or the pair is irrational but can be written as the sum of a common
irrational number a and different rational numbers (in which case C 00 ⊆ Ba , which is not maximal if it is a proper subset).
We’re left then with some subset of R where some a and b are incomparable, so C 00 is not simply-ordered. We conclude
that the collection of all Ba constitutes all the maximal simply-ordered subsets of R.

Problem 11.3. [See question.]

Solution: This process will work for example 2. Let (x0 , y0 ) ∈ R2 be the generator of B. If (x1 , y1 ) ∈ R2 is distinct from
(x0 , y0 ), it will be an element of B only if y0 = y1 . Since x0 < x1 or x1 < x0 , the two points are comparable. Therefore
B is a maximal simply-ordered subset of R2 .
This process will not work for example 1. Suppose A1 = {(x, y) ∈ R2 : x2 + y 2 ≤ 16}, A2 = {(x, y) ∈ R2 :
(x − 2)2 + y 2 ≤ 1} and A3 = {(x, y) ∈ R2 : (x + 2)2 + y 2 ≤ 1}. These nonempty sets correspond to a circular region
of radius 4 centered at the origin, a circular region of radius 1 centered at (2, 0), and circular region of radius 1 centered
at (−2, 0). We have A2 ⊂ A1 and A3 ⊂ A1 , so B = {A2 , A3 }. But B is not simply ordered because A2 ∩ A3 = ∅. The
process has failed!

Problem 11.4. Given two points (x0 , y0 ) and (x1 , y1 ) of R2 , define (x0 , y0 ) ≺ (x1 , y1 ) if x0 < x1 and y0 ≤ y1 . Show that
the curves y = x3 and y = 2 are maximal simply ordered subsets of R2 , and the curve y = x2 is not. Find all maximal simply
ordered subsets.

Solution: We’ll first establish the universe of maximal simply-ordered subsets of R. The answers for the three functions at
issue will immediately follow.
General Proposition. Let C be the collection of maximal simply-ordered subsets of R2 . We’ll see every element of C
is a set of points in R2 of some increasing function on some subset of R whose domain or codomain is unbounded (possibly
both). Let C be a maximal simply-ordered set of R2 . Any distinct p0 = (x0 , y0 ), p1 = (x1 , y1 ) ∈ C are comparable. If we
assume without loss of generality that p0 ≺ p1 , then x0 < x1 and y0 ≤ y1 . Further, it cannot be that x0 < x1 and y0 > y1
because then p0 and p1 would be incomparable. We infer that the elements of C correspond to an increasing function on
some subset of R.
Suppose the x- and y- components of C’s elements are bounded by Mx and My (i.e, |x| ≤ Mx and |y| ≤ My for all
(x, y) ∈ C). It follows that q = (−2Mx , −2My ) ≺ (x, y) for all (x, y) ∈ C. Consequently, C ∪ {q} is simply-ordered,
contradicting that C is maximal. It follows that the increasing function associated with C must have an unbounded domain
or an unbounded codomain..
Now suppose f : S → R is an increasing function where S ⊆ R and whose domain or codomain is unbounded. Let
C 0 = {(x, f (x)) ⊂ R2 }. By definition, if x0 < x1 , then f (x0 ) ≤ f (x1 ). Consequently, for any p0 , p1 ∈ C 0 , either p0 ≺ p1 or
p1 ≺ p0 , so C 0 is simply ordered. Assume that C 0 is a proper subset of D ⊆ R2 where (x2 , y2 ) ∈ D\C 0 . Clearly y2 6= f (x2 ).
If x2 in contained in f ’s domain S, there is some (x2 , f (x2 )) ∈ C 0 . Since the two points are distinct, (x2 , f (x2 )) is not
comparable with (x2 , y2 ). On the other hand, if x2 ∈ / S (in which case f ’s domain is bounded), it follows that f ’s codomain
is unbounded and x2 is either to the left or right of every point in S. If x2 is to the right of S, there is some x3 < x2 in
S where f (x3 ) > y2 because f ’s codomain tends toward ∞. If x2 is to the left of S, there is some x3 > x2 in S where
f (x3 ) < y2 since f ’s codomain tends toward −∞. Either way, (x2 , y2 ) is not comparable with (x3 , f (x3 )). We conclude
that C 0 is a maximal simply-ordered subset of R2 and therefore belongs to C.
As a result, C is the collection of the sets of the points of increasing functions on some subset R with unbounded
domains or codomains.
Functions at Issue. We see that A1 = {(x, y) ∈ R2 : y = x3 } corresponds to an increasing function with an unbounded
domain and codomain. The same is true for {(x, y) ∈ R2 : y = 2}. Accordingly, A1 and A2 are maximal simply-ordered

Page 14
subsets of R2 . On the other hand, A3 = {(x, y) ∈ R2 : y = x2 } is not increasing on R. Therefore A3 is not a maximal
simply-ordered subset of R2 .

Problem 11.5. Show that Zorn’s lemma implies the following: Lemma (Kuratowski). Let A be a collection of sets. Suppose
that for every subcollection B that is simply ordered by proper inclusion, the union of the elements of B belongs to A. Then A
has an element that is properly contained in no other element of A.

Solution: Let ≺ be proper inclusion on A, which


S we have seen is a strict partial ordering. Suppose B is a subcollection
of A that is simply ordered by ≺ and Bup = B∈B ∈ A. It follows that if B ∈ B, then B is a subset of Bup , so either
B ≺ Bup or B = Bup . By definition, Bup is an upper bound of B. Since B is arbitrary, every such subcollection B has
an upper bound. Zorn’s Lemma implies that A has a maximal element Amax . As a result, there is no A ∈ A such that
Amax ≺ A. It follows that Amax is not properly contained in any element of A.
S
NB: Just because B is simply-ordered by inclusion does not mean that B∈B B is in B, even though I thought at
first (mistakingly) that this was the case. Here is a simple counterexample. Consider the collection U of all finite
sections of natural numbers. It follows that U = {S1 , S2 , S3 , . . .} and is infinite. The union of all sets in U equals
N, which is not contained in U.

Problem 11.6. A collection A of subsets of a set X is said to be of finite type provided that a subset B of X belongs to A
if and only if every finite subset of B belongs to A . Show that the Kuratowski lemma implies the following: Lemma (Tukey,
1940). Let A be a collection of sets. If A is of finite type, then A has an element that is properly contained in no other element
of A.

Solution: Suppose A is a collection of sets that is of finite type. Let ≺ beSa strict partial ordering of A defined by proper
inclusion, and let B be a simply-ordered subcollection of A. Now let Bu = B∈B , and let Bf be a finite subset of Bu . For
each bi ∈ Bu (of which there are a finite number), there is some Bi ∈ B where bi ∈ Bi . But since B is simply-ordered,
there must be some B0 ∈ B where Bf is subset of B0 (if that weren’t the case and some set of bi were in one set in B and
a disjoint set of bi were in another set in B, the two sets would be incomparable). Since A is of finite type, every finite
subset of B0 must be in A, so Bf ∈ A. Consequently, every finite subset of Bu is in A, so Bu ∈ A. From Kuratowski’s
Lemma, A has an element that is not properly contained in any other element of A.

Problem 11.8. [See question.]

Solution: Part (a) Suppose V is a vector space over the field K and A is a proper subset of V where A is independent.
Let v ∈ V where v is not in A’s span. By definition, there is no linear combination of the vectors in A that will equal v. It
follows that there is also no linear combination of the vectors in A that equal cv for any non-zero c ∈ K (if there were, you
could divide each of the coefficients of the vectors in A by c and get a linear combination that equals v). Now suppose:
X
wu u + cv = 0,
u∈A

for some wu , c ∈ K. In order for this relationship to hold, the summation on the left must equal −cv. As shown above,
this is only possible if c = 0. Since A is independent, the summation will equal 0 only if each wu = 0. As a result, if:
X
wu0 u0 = 0,
u0 ∈A∪{v}

then each wu0 = 0. Therefore A ∪ {v} is independent.


Part (b) Let C be the collection of all independent subsets of V . Define ≺ as a relation on C where if C0 , C1 ∈ C,
then C0 ≺ C1 if C0 is a proper subset of C1 . This is a strict partial ordering on C. Observe that C0 ≺ C0 is impossible
because no nonempty set is its own proper subset. If C2 ∈ C and C0 ≺ C1 and C1 ≺ C2 , then C0 is a proper subset of
C2 , so C0 ≺ C2 .
Now suppose A is an element of C, in which case A is independent. Let A0 be any finite subset of A. Since any linear
combination of A equals zero only if it is trivial (i.e., the coefficient of each element of A is zero), it follows that the same

Page 15
is true for A0 , so A0 is independent. Consequently, A0 is an element of C. Conversely, suppose D is a subset of V and
every finite subset D0 of D is in C. It follows that each D0 is independent, so any linear combination of the elements of
D0 that equals zero is the trivial case. Since this is true for every finite subset of D, every finite linear combination of
vectors in D equal to zero. (Note that independence requires that the ”only finite linear combination of elements of [D]
that equals the zero vector is the trivial one having all coefficients zero.” Since every finite subset of D is independent, this
is necessary true.)
We infer that B is independent and therefore contained in C. To sum up, a subset of V is in C if and only if every one
of its finite subsets is in C. Accordingly, C is of finite type. By Tukey’s Lemma, C has a maximal element.
Part (c) Let B be the maximal element of C. The set B must be independent. Moreover, B must span V ; otherwise,
for any v ∈ V that is not in B’s span, B ∪ {v}, would be independent, contradicting that B is maximal. Hence, B is a
basis of V .

Page 16

You might also like